Course Content
All Previous Years Krok 2 Papers with Explanations
About Lesson

Question From (1 To 50 )

  1. A 27-year-old man received an electrical injury, when he touched an exposed electrical wire with his hand, after which he developed circulatory and respiratory arrest. Resuscitation measures restored his cardiac activity after 5 minutes. What complications are possible in a few hours or even days after the electrical injury?

A. Circulatory arrest

B. Acute renal failure

C. Respiratory arrest

D. Pulmonary edema

E. Acute liver failure


Answer:  Circulatory arrest

Explanation

While circulatory arrest is an immediate complication of electrical injury, several other complications can occur in the hours or days following the injury. However, among the options provided, the most likely complication to occur after the electrical injury and resuscitation is D. Pulmonary edema.  

Electrical injury can cause damage to the lungs and pulmonary vasculature, leading to the development of pulmonary edema (fluid accumulation in the lungs). This can occur as a result of the direct effect of the electrical current on the lungs, or as a secondary effect of circulatory and respiratory arrest and subsequent resuscitation measures.  

Acute renal failure (B) and acute liver failure (E) are possible complications of electrical injury, but are less common than pulmonary edema and typically occur in more severe cases.   Respiratory arrest (C) is already mentioned as an immediate complication in the case description, so it wouldn’t be a possible complication in the hours or days following the injury.   Therefore, the most appropriate answer to this question is D. Pulmonary edema.


2. Survey X-ray of the patient’s abdominal cavity has shown several hemispherical areas of lucency, located above welldefined horizontal levels. What might be the cause of such an X-ray presentation?

A. Intestinal obstruction

B. Cancer of the large intestine

C. Meteorism

D. Perforated ulcer

E. —


Answer: Intestinal obstruction

Explanation

The most likely cause of an X-ray presentation showing several hemispherical areas of lucency, located above well-defined horizontal levels, is A. Intestinal obstruction.   Intestinal obstruction is a condition in which there is a blockage of the normal flow of intestinal contents through the gastrointestinal tract.

This can occur as a result of a mechanical obstruction, such as a tumor, adhesion, or hernia, or a functional obstruction, such as in cases of paralytic ileus. X-ray findings in cases of intestinal obstruction can include air-fluid levels, which appear as hemispherical areas of lucency, located above well-defined horizontal levels.  

Cancer of the large intestine (B) can cause changes in bowel habits and abdominal pain, but is less likely to produce the specific X-ray findings described.   Meteorism (C), or excessive gas accumulation in the gastrointestinal tract, can also cause abdominal distension and discomfort, but is less likely to produce the specific X-ray findings described.  

Perforated ulcer (D) can cause acute abdominal pain and tenderness, and can lead to the development of peritonitis. However, the specific X-ray findings described are less likely to be seen in cases of perforated ulcer.   Option E is not provided.   Therefore, the most appropriate answer to this question is A. Intestinal obstruction.


3. A patient has visited the doctor at a rural outpatient clinic twice during the calendar year (in March and in November). In both cases, he was diagnosed with acute respiratory viral infection. How to fill in the statistical forms for registration of final (clarified) diagnoses, to account for these morbidity cases?

A. For each case separately with the (+) sign

B. For each case separately with the (–) sign

C. For the first case with the (+) sign, for the second case with the (–) sign

D. The forms are not filled for either of these cases

E. For the first case with the (+) sign, for the second case no form is necessary


Answer: For each case separately with the (+) sign

Explanation

The appropriate way to fill in the statistical forms for registration of final (clarified) diagnoses for the patient who visited the doctor twice during the calendar year with acute respiratory viral infection is to choose A. For each case separately with the (+) sign.   Each case should be recorded separately with a positive sign, as both episodes of acute respiratory viral infection are considered distinct cases of illness.

This approach allows for the accurate accounting of the number of cases of illness and the calculation of morbidity rates.   Option B, for each case separately with the (-) sign, would be inappropriate as this would indicate that the patient did not have acute respiratory viral infection, which is not true.  

Option C, for the first case with the (+) sign, for the second case with the (-) sign, would also be inappropriate as both cases represent distinct episodes of acute respiratory viral infection.   Option D, the forms are not filled for either of these cases, would be inappropriate as it would result in underreporting of morbidity and inaccurate tracking of disease trends.  

Option E, for the first case with the (+) sign, for the second case no form is necessary, would also be inappropriate as both cases should be recorded separately to accurately track the incidence and prevalence of disease.


4. A 55-year-old man is complaining of general weakness, decreased fluid excretion, and an aching pain in his heart. For the last 15 years he has been suffering from chronic pyelonephritis and undergoing an inpatient treatment. His skin is dry and has a yellowish tinge. His pulse is 80/min., rhythmic, blood pressure – 100/70 mm Hg. Cardiac auscultation detects muffled heart sounds and a friction rub in the pericardium. Laboratory results: creatinine – 1.1 mmol/L, glomerular filtration – 5 mL/min. What treatment is indicated for this patient?

A. Hemodialysis

B. Diuretics

C. Rheopolyglucin (Dextran)

D. Xylit, Sorbitol

E. Antibiotics


Answer: Hemodialysis

Explanation

The most appropriate treatment for this patient, based on the provided information, is A. Hemodialysis.   The patient’s symptoms and laboratory results suggest that he is experiencing advanced renal failure, likely as a complication of his chronic pyelonephritis. The presence of a friction rub in the pericardium suggests that he may also have developed uremic pericarditis.  

Hemodialysis is the primary treatment for advanced renal failure and can help to remove excess fluid and waste products from the body. Hemodialysis is indicated for patients with a glomerular filtration rate (GFR) of less than 15 mL/min, which is the case for this patient.  

Diuretics (B) may be used to help manage fluid overload in patients with renal failure, but would not be sufficient as a standalone treatment for this patient.   Rheopolyglucin (C) and Xylit, Sorbitol (D) are not indicated for the treatment of advanced renal failure.   Antibiotics (E) may be indicated for the treatment of acute pyelonephritis, but would not be appropriate as a standalone treatment for advanced renal failure.  

Therefore, the most appropriate treatment for this patient is A. Hemodialysis.


5. After lifting a heavy object, a 41-yearold man felt a sharp pain in his chest on the right and developed progressing shortness of breath. The man’s condition is severe. His lips and mucosa are cyanotic, respiratory rate – 28/min., pulse – 122/min. Over the right half of the chest, percussion shows tympanitis, while auscultation detects sharply weakened breathing. The II heart sound is accentuated over the pulmonary artery. His blood pressure is 80/40 mm Hg. What primary emergency measure must be taken at the prehospital stage in this case?

A. Aspiration of air from the pleural cavity

B. Administration of euphyllin (aminophylline)

C. Call for a cardiology team

D. Administration of adrenaline

E. Oxygen inhalation


Answer: Aspiration of air from the pleural cavity

Explanation

The most appropriate primary emergency measure that must be taken at the prehospital stage for this patient is A. Aspiration of air from the pleural cavity.   The patient’s symptoms and physical examination findings suggest the presence of a tension pneumothorax, which can be a life-threatening condition. Tension pneumothorax can occur after trauma or injury to the chest, such as after lifting a heavy object. In this condition, air enters the pleural cavity and becomes trapped, causing compression of the lung and resulting in respiratory distress and cardiovascular compromise.  

Aspiration of air from the pleural cavity is an essential emergency intervention for tension pneumothorax. This involves inserting a needle or catheter into the chest to release the trapped air and relieve the pressure on the lung and surrounding structures.   Administration of euphyllin (B), adrenaline (D), or oxygen inhalation (E) may be helpful in managing respiratory distress and cardiovascular compromise but are not the primary emergency measure for this condition.  

Call for a cardiology team (C) would not be the most appropriate initial action in this case, as the patient’s symptoms and physical examination findings suggest a respiratory emergency rather than a cardiac emergency.   Therefore, the most appropriate primary emergency measure that must be taken at the prehospital stage for this patient is A. Aspiration of air from the pleural cavity.


6. For a month a 60-year-old patient had short-term episodes of decreased strength in his limbs on the left. Later, after waking up in the morning, he developed a persistent weakness in his extremities. His blood pressure is 140/90 mm Hg. He is conscious and has central paresis of VII and XII pairs of cranial nerves on the left. On the same side, he presents with central hemiparesis and hemihyperesthesia. What group of drugs must be chosen for the differentiated treatment of this patient?

A. Anticoagulants

B. Hypotensive agents

C. Diuretics

D. Corticosteroids

E. Hemostatics


Answer: Anticoagulants

Explanation

The most appropriate group of drugs for the differentiated treatment of this patient, based on the provided information, is A. Anticoagulants.   The patient’s symptoms and physical examination findings suggest that he may have had a stroke, likely due to a thrombus or embolism blocking blood flow to the brain.

The presence of central paresis of cranial nerves VII and XII on the left side, as well as central hemiparesis and hemihyperesthesia, further support this diagnosis.   Anticoagulants, such as heparin and warfarin, are indicated for the prevention of thromboembolic events, including stroke.

These medications can help to prevent the formation of blood clots and reduce the risk of further stroke.   Hypotensive agents (B) and diuretics (C) may be indicated for the management of hypertension, which is a risk factor for stroke, but would not be appropriate as a standalone treatment for stroke.  

Corticosteroids (D) are not indicated for the treatment of stroke and may actually worsen outcomes in some cases.   Hemostatics (E) are not indicated for the treatment of stroke and may actually increase the risk of bleeding complications.   Therefore, the most appropriate group of drugs for the differentiated treatment of this patient is A. Anticoagulants.


7. A 45-year-old woman with a normal body weight was diagnosed with diabetes mellitus for the first time. Attempts to correct his blood glucose levels by means of a diet were unsuccessful. Her 24- hour glycemia varies between 10 and 15 mmol/L. Which medicine would be optimal in this case?

A. Sulfanilamide hypoglycemic drugs

B. Biguanides combined with sulfanilamide hypoglycemic drugs

C. Insulin

D. Insulin combined with sulfanilamide hypoglycemic drugs

E. Biguanides 922521I1 2


Answer:  Sulfanilamide hypoglycemic drugs

Explanation

The most appropriate medicine for the treatment of a 45-year-old woman with newly diagnosed diabetes mellitus and elevated blood glucose levels despite dietary interventions would be A. Sulfanilamide hypoglycemic drugs.   Sulfanilamide hypoglycemic drugs, also known as sulfonylureas, are a class of oral antidiabetic medications that stimulate the release of insulin from the pancreas and improve insulin sensitivity in peripheral tissues. These medications can be effective in controlling blood glucose levels in patients with type 2 diabetes, particularly in the early stages of the disease.  

Biguanides (E) such as metformin are another class of oral antidiabetic medications that can be effective in controlling blood glucose levels in patients with type 2 diabetes. However, they are typically used as a first-line treatment in patients who are overweight or obese, and may be less effective in patients with normal body weight.   Insulin (C) is typically reserved for the treatment of more advanced or poorly controlled diabetes, or in cases where oral antidiabetic medications are ineffective or contraindicated.  

Combination therapy with sulfanilamide hypoglycemic drugs and biguanides (B) or insulin (D) may be considered in some cases, but would not be the optimal first-line treatment for this patient.   Therefore, the most appropriate medicine for the treatment of a 45-year-old woman with newly diagnosed diabetes mellitus and elevated blood glucose levels despite dietary interventions would be A. Sulfanilamide hypoglycemic drugs.


8. Fluorography of a 45-year-old patient detects a few foci of low intensity with blurred margins on the apex of his right lung. This sign is observed for the first time. Such condition causes him no discomfort. He has a many-year history of smoking. Objectively, percussion produces a pulmonary sound above the lungs, the respiration is vesicular, auscultation detects no wheezing. Blood test findings are normal. Make the right diagnosis:

A. Focal pulmonary tuberculosis

B. Peripheral lung cancer

C. Bronchopneumonia

D. Disseminated pulmonary tuberculosis

E. Eosinophilic pneumonia


Answer: Focal pulmonary tuberculosis

Explanation

The most likely diagnosis for the 45-year-old patient with a few foci of low intensity with blurred margins on the apex of his right lung detected on fluorography is A. Focal pulmonary tuberculosis.  

The patient’s history of smoking and the presence of new pulmonary foci on imaging are concerning for a possible diagnosis of tuberculosis, which is a common respiratory infection that can be associated with smoking. The absence of wheezing and normal blood test findings suggest that the patient’s condition is not due to bronchopneumonia (C) or eosinophilic pneumonia (E).  

Peripheral lung cancer (B) is a possibility, but would typically present with more specific imaging findings and may be associated with other symptoms such as cough, weight loss, or hemoptysis.   Disseminated pulmonary tuberculosis (D) typically involves multiple areas of the lungs and may be associated with extrapulmonary symptoms.  

Therefore, the most likely diagnosis for the patient’s condition is A. Focal pulmonary tuberculosis. Further testing, such as sputum analysis or a chest CT scan, may be necessary to confirm the diagnosis and guide treatment.


9. A 54-year-old patient has been suffering from femoral osteomyelitis for over 20 years. In the last month she developed gradually progressing leg edemas. In the urine: proteinuria – 6.6 g/L. In the blood: dysproteinemia in the form of hypoalbuminemia, increased α2- and γglobulin levels, ESR – 50 mm/hour. Make the right diagnosis:

A. Secondary renal amyloidosis

B. Acute glomerulonephritis

C. Systemic lupus erythematosus

D. Chronic glomerulonephritis

E. Multiple myeloma


Answer: Secondary renal amyloidosis

Explanation

The most likely diagnosis for the 54-year-old patient with femoral osteomyelitis, leg edema, proteinuria, dysproteinemia, and an elevated ESR is A. Secondary renal amyloidosis.   Secondary renal amyloidosis is a complication of chronic inflammatory conditions, such as chronic infections or inflammatory diseases like osteomyelitis. In this condition, abnormal protein deposits called amyloid accumulate in the kidneys, leading to renal dysfunction and proteinuria. The presence of hypoalbuminemia, increased α2- and γ-globulin levels, and an elevated ESR further support this diagnosis.  

Acute glomerulonephritis (B) typically presents with symptoms such as hematuria, proteinuria, and hypertension, and may be associated with a recent streptococcal infection.   Systemic lupus erythematosus (C) is an autoimmune disease that can affect multiple organs, including the kidneys, and may present with a variety of symptoms such as joint pain, rash, and proteinuria.   Chronic glomerulonephritis (D) is a progressive inflammatory disease that can lead to renal failure, and may present with proteinuria, hypertension, and renal dysfunction.  

Multiple myeloma (E) is a cancer of the plasma cells that can lead to bone destruction and renal dysfunction, but would typically present with other symptoms such as bone pain and anemia.   Therefore, the most likely diagnosis for the patient’s condition is A. Secondary renal amyloidosis. Treatment may involve management of the underlying chronic inflammatory condition and supportive care for renal dysfunction.


10. A 28-year-old man was hospitalized on the 9th day of illness with complaints of fever of 39oC, headache, general weakness, constipation, and disturbed sleep. On the skin of his abdomen there were isolated roseolas, his pulse is 78/min., the liver is enlarged by 2 cm. Make the right diagnosis:

A. Typhoid fever

B. Brucellosis

C. Leptospirosis

D. Sepsis

E. Epidemic typhus


Answer: Typhoid fever

Explanation

The most likely diagnosis for the 28-year-old man with fever, headache, general weakness, constipation, and skin roseolas, along with an enlarged liver, is A. Typhoid fever.   Typhoid fever is a bacterial infection caused by Salmonella enterica serotype Typhi, which is transmitted through contaminated food or water.

The symptoms typically include fever, headache, abdominal pain, constipation or diarrhea, and a characteristic rash of rose-colored spots on the trunk of the body. Enlargement of the liver and spleen is also common.   Brucellosis (B) is a bacterial infection typically transmitted through contact with infected animals or consumption of contaminated dairy products. Symptoms may include fever, headache, joint pain, and fatigue.  

Leptospirosis (C) is a bacterial infection typically transmitted through contact with contaminated water or soil. Symptoms may include fever, headache, muscle pain, and jaundice.   Sepsis (D) is a potentially life-threatening condition that can occur as a complication of any type of infection, and may present with symptoms such as fever, rapid heart rate, and low blood pressure.  

Epidemic typhus (E) is a bacterial infection typically transmitted through contact with infected body lice. Symptoms may include fever, headache, and a rash, but enlargement of the liver and spleen is typically not present.   Therefore, the most likely diagnosis for the patient’s condition is A. Typhoid fever. Treatment typically involves antibiotics and supportive care for symptom management.


11. A well was dug on a rural land plot. It is located at the distance of 20 meters from the house, 10 meters from the toilet, and 15 meters from the neighbor’s house. What is the minimum permissible distance between the well and the source of possible water pollution, according to the sanitary norms?

A. 30 meters

B. 25 meters

C. 20 meters

D. 15 meters

E. 10 meters


Answer: 30 meters

Explanation

The minimum permissible distance between the well and a potential source of water pollution according to sanitary norms varies depending on the type of pollution source. However, in general, the minimum distance is typically 30 meters (100 feet).   In this case, the well is located 20 meters from the house, 10 meters from the toilet, and 15 meters from the neighbor’s house.

While there may be other factors to consider, such as the type of soil and the presence of other potential sources of pollution, it is likely that the minimum permissible distance of 30 meters would not be met in this situation.   Therefore, the minimum permissible distance between the well and the source of possible water pollution, according to the sanitary norms, is A. 30 meters.


12. The ambulance has brought in the victim of an accident, who has fallen from a height. This patient has clinical signs of multiple fractures in both legs. The patient’s condition is severe; the patient is conscious, but mentally sluggish; the skin is pale gray; the forehead is covered in a cold sweat. The patient’s breathing is shallow, with respiratory rate of 30/min., blood pressure – 80/60 mm Hg, pulse – 120/min., shallow. What complication has occurred in this person?

A. Traumatic shock, degree 2

B. Traumatic shock, degree 1

C. Traumatic shock, degree 4

D. Syncope

E. Traumatic shock, degree 3


Answer: Traumatic shock, degree 2

Explanation

The most likely complication that has occurred in the victim of an accident who has fallen from a height, with multiple fractures in both legs, is A. Traumatic shock, degree 2.   Traumatic shock is a condition that can occur as a result of severe injury or trauma, and is characterized by a combination of hypovolemia (low blood volume), tissue damage, and poor tissue perfusion.

The symptoms of traumatic shock may include mental confusion or sluggishness, pale and clammy skin, shallow breathing, rapid heart rate, and low blood pressure.   In this case, the patient’s symptoms of mental sluggishness, pale gray skin, cold sweat, shallow breathing, and tachycardia with shallow pulse suggest the presence of traumatic shock. The severity of the shock can be classified into four degrees, with degree 2 indicating moderate shock.  

Syncope (D) is a sudden loss of consciousness that may be due to a variety of causes, but is less likely in this case given the patient’s other symptoms.   Therefore, the most likely complication that has occurred in this person is A. Traumatic shock, degree 2. Treatment may involve fluid resuscitation, pain management, and stabilization of the fractures.


13. A 30-year-old patient after an emotional upset developed tonic seizure that later gave place to clonic seizures. This condition was accompanied by loss of consciousness, foaming at the mouth, and involuntary urination. No focal signs were detected. Blood pressure is 120/60 mm Hg. Make the right diagnosis:

A. Epilepsy

B. Syncope

C. Neurasthenia

D. Eclampsia

E. Stroke


Answer: Epilepsy

Explanation

The most likely diagnosis for the 30-year-old patient who developed tonic seizure that later gave place to clonic seizures after an emotional upset, accompanied by loss of consciousness, foaming at the mouth, and involuntary urination, is A. Epilepsy.   Epilepsy is a neurological disorder characterized by recurrent seizures.

Seizures may be triggered by a variety of factors, including emotional stress. The symptoms described in the scenario, including tonic-clonic seizures, loss of consciousness, foaming at the mouth, and involuntary urination, are consistent with a diagnosis of epilepsy.   Syncope (B) is a sudden loss of consciousness that may be due to a variety of causes, but typically does not involve seizures or foaming at the mouth.   Neurasthenia (C) is a term that was historically used to describe a condition of nervous exhaustion, but is not currently recognized as a diagnostic category.  

Eclampsia (D) is a complication of pregnancy characterized by seizures and high blood pressure, and would not be a likely diagnosis in a non-pregnant patient.   Stroke (E) is a neurological emergency caused by a disruption of blood flow to the brain, and typically presents with symptoms such as weakness or numbness on one side of the body, difficulty speaking, and vision changes.   Therefore, the most likely diagnosis for the patient’s condition is A. Epilepsy. Further testing, such as an EEG or brain imaging, may be necessary to confirm the diagnosis and guide treatment.


14. A full-term delivery has resulted in the birth of a live full-term girl without asphyxia. The baby is inert, her skin is pale with an icteric tinge, she has no edemas. The abdomen is soft, the liver and spleen are enlarged. The blood tests show that the mother’s blood type is A(II) Rh(–), while the child’s blood type is A(II) Rh(+). Make the right diagnosis:

A. Hemolytic disease of the newborn

B. Intracranial birth injury

C. Physiological jaundice

D. Maldevelopment of parenchymal organs

E. Disturbed cerebral circulation


Answer: Hemolytic disease of the newborn          

Explanation

The most likely diagnosis for the full-term girl who was born without asphyxia, but has pale skin with an icteric tinge, enlarged liver and spleen, and a mother with blood type A(II) Rh(–) and a child with blood type A(II) Rh(+) is A. Hemolytic disease of the newborn.  

Hemolytic disease of the newborn (HDN) is a condition that occurs when there is an incompatibility between the blood types of the mother and the fetus, leading to the destruction of the fetal red blood cells. In this case, the mother has Rh-negative blood and the baby has Rh-positive blood, which can result in the production of antibodies by the mother that can attack the baby’s red blood cells. The symptoms of HDN can include pale skin with an icteric tinge, enlarged liver and spleen, anemia, and jaundice.  

Intracranial birth injury (B) may occur if there is trauma or damage to the baby’s head during delivery, but would typically present with symptoms such as seizures or neurological deficits.   Physiological jaundice (C) is a common condition that can occur in newborns due to the normal breakdown of red blood cells and is not typically associated with enlarged liver and spleen.   Maldevelopment of parenchymal organs (D) is a broad category that can include a variety of congenital anomalies, but would typically present with more specific symptoms depending on the affected organ.  

Disturbed cerebral circulation (E) may occur in cases of birth asphyxia or other conditions that affect blood flow to the brain, and would typically present with neurological symptoms such as seizures or altered mental status.   Therefore, the most likely diagnosis for the baby’s condition is A. Hemolytic disease of the newborn. Treatment may involve phototherapy to manage jaundice, blood transfusions to treat anemia, and close monitoring for potential complications.


15. The region in the Carpathian foothills is characterized with constant high atmospheric humidity (over 80%). In the cold season, while the air temperature is moderately low, the population of this region feels extreme cold. What type of heat transfer increases in such conditions?

A. Convection

B. –

C. Evaporation

D. Radiation

E. Conduction


Answer: Convection

Explanation

The type of heat transfer that increases in conditions of constant high atmospheric humidity and moderately low air temperature in the Carpathian foothills region is A. Convection.   Convection is heat transfer due to the movement of fluids (such as air or water) caused by differences in temperature and density. In regions with high atmospheric humidity, the moisture in the air can increase the heat loss from the body through convection, as the moisture in the air can carry heat away from the body more efficiently than dry air.

In addition, in moderately low air temperatures, the body may lose heat more rapidly through convection as the temperature difference between the body and the surrounding air increases.   Evaporation (C) is another type of heat transfer that can contribute to cooling of the body, but is less likely to be a significant factor in cold and humid conditions.  

Radiation (D) is heat transfer through electromagnetic waves and is not affected by atmospheric humidity.   Conduction (E) is heat transfer through direct contact and is not likely to be a significant factor in this scenario, as the body is not in direct contact with cold surfaces.   Therefore, the most likely type of heat transfer that increases in these conditions is A. Convection.


16. A 58-year-old patient complains of 922521I1 3 an inguinal tumor that increases during straining and coughing. There is a pulsating tumor-like formation located below and laterally to the Poupart’s ligament. The formation does not diminish during palpation. Make the right diagnosis:

A. Femoral aneurysm

B. Femoral hernia

C. Neurinoma of the femoral nerve

D. Ectopic testicle

E. Inguinal hernia


Answer: Femoral aneurysm

Explanation

The most likely diagnosis for the 58-year-old patient who complains of an inguinal tumor that increases during straining and coughing, with a pulsating tumor-like formation located below and laterally to the Poupart’s ligament that does not diminish during palpation, is A. Femoral aneurysm.   A femoral aneurysm is a bulging of the wall of the femoral artery in the groin area, which can cause a pulsating mass that may increase in size during straining or coughing. The mass is typically located below and lateral to the inguinal ligament and does not diminish during palpation.  

Femoral hernia (B) is another possible diagnosis in this location, but would typically present as a soft, reducible mass that may disappear when the patient lies down.   Neurinoma of the femoral nerve (C) is a rare type of nerve tumor that can cause pain or weakness in the thigh, but would not typically present as a pulsating mass.  

Ectopic testicle (D) is a rare condition in which the testicle fails to descend into the scrotum, but would not typically present as a pulsating mass in the groin area.   Inguinal hernia (E) is another possible diagnosis in this location, but would typically present as a soft, reducible mass that may disappear when the patient lies down.   Therefore, the most likely diagnosis for the patient’s condition is A. Femoral aneurysm. Further testing, such as ultrasound or CT scan, may be necessary to confirm the diagnosis and guide treatment.


17. A variety of lighting fixtures have been used to illuminate classrooms. What kind of lighting fixtures creates the most hygienically acceptable lighting?

A. Lighting fixtures that provide reflected lighting

B. Lighting fixtures that provide direct lighting

C. Lighting fixtures that provide scattered lighting

D. Lighting fixtures that provide semireflected lighting

E. Lighting fixtures that provide combined lighting


Answer: Lighting fixtures that provide reflected lighting

Explanation

The lighting fixtures that create the most hygienically acceptable lighting are A. Lighting fixtures that provide reflected lighting.   Reflected lighting is a type of lighting in which the light is directed towards the ceiling or walls, and then reflected back down onto the work surface. This type of lighting provides a more diffuse and uniform illumination, which can reduce glare and shadows, and is less likely to cause eye strain or other visual discomfort.   Direct lighting (B) provides illumination directly onto the work surface, which can create shadows and increase the risk of glare and visual discomfort.  

Scattered lighting (C) is a type of lighting in which the light is dispersed in many directions, which can create a diffused and soft illumination, but may also increase the risk of glare and visual discomfort.   Semireflected lighting (D) is a type of lighting that combines direct and reflected light, which can provide a balance between illumination and visual comfort, but may also create shadows and increase the risk of glare.  

Combined lighting (E) is a general term that can refer to any type of lighting that combines two or more lighting techniques, and is not specific to any particular type of lighting fixture.   Therefore, the most hygienically acceptable lighting fixtures are those that provide reflected lighting.


18. A victim of a car accident received multiple fractures of the limbs and pelvic bones. He has a history of hemophilia A. Objectively, hematomas form at the sites of injury. The patient’s condition is deteriorating. The blood pressure is 90/50 mm Hg. What is the optimal combination of infusion agents that should be given to the patient for treatment after polyglucin (dextran) and saline solutions are administered?

A. Cryoprecipitate, packed erythrocytes

B. Packed erythrocytes, fresh frozen plasma

C. Fresh frozen plasma, albumin

D. Cryoprecipitate, glucose

E. Packed erythrocytes


Answer:  Cryoprecipitate, packed erythrocytes

Explanation

The optimal combination of infusion agents that should be given to the patient for treatment after polyglucin (dextran) and saline solutions are administered is A. Cryoprecipitate, packed erythrocytes.   The patient has a history of hemophilia A, which is a bleeding disorder caused by a deficiency in clotting factor VIII.

Multiple fractures and hematomas can further exacerbate the bleeding, leading to a worsening of the patient’s condition.   Cryoprecipitate contains concentrated clotting factors, including factor VIII, which can help to stop bleeding in patients with hemophilia A. Packed erythrocytes can help to replenish the patient’s red blood cell count, which may be depleted due to bleeding.   Fresh frozen plasma (B) contains all of the clotting factors, but may not be as effective in treating hemophilia A as cryoprecipitate, which contains a higher concentration of factor VIII.  

Albumin (C) is a protein that can help to maintain blood volume, but would not be effective in treating bleeding disorders such as hemophilia A.   Glucose (D) is a type of sugar that is used to provide energy to the body, but would not be effective in treating bleeding disorders.  

Packed erythrocytes (E) alone may not be sufficient in treating the patient’s bleeding disorder, as they do not contain clotting factors.   Therefore, the optimal combination of infusion agents for the patient’s condition is A. Cryoprecipitate, packed erythrocytes. Close monitoring of the patient’s condition and ongoing treatment may be necessary to manage potential complications of the bleeding disorder and fractures.


19. A 40-year-old woman, gravida 6, para 3, at the 40 weeks of her pathological pregnancy (threatened miscarriage, type I gestosis of the second half of her pregnancy), gave birth to a girl with asphyxia. The baby’s condition is severe, the weight is 2 kg, there are signs of immaturity and hydrocephalus. The baby’s skin is pale, icteric, and has acrocyanosis. Heart sounds are muffled, there is a harsh systolic murmur at all points of auscultation. Abdomen is enlarged, the liver is +3 cm. The urine is saturated, the stool is light-colored. An ophthalmologist has detected chorioretinitis in the baby. Make the right diagnosis:

A. Congenital toxoplasmosis

B. Hemolytic disease of the newborn

C. Sepsis

D. Congenital heart defect

E. Congenital hepatitis


Answer: Congenital toxoplasmosis

Explanation

The most likely diagnosis for the 40-year-old woman’s newborn girl with asphyxia, signs of immaturity and hydrocephalus, pale and icteric skin with acrocyanosis, muffled heart sounds with a harsh systolic murmur, enlarged abdomen and liver, saturated urine, light-colored stool, and chorioretinitis is A. Congenital toxoplasmosis.  

Congenital toxoplasmosis is a condition that occurs when a pregnant woman becomes infected with the Toxoplasma gondii parasite, which can be transmitted to the fetus through the placenta. Infection during pregnancy can cause a range of symptoms in the newborn, including neurological abnormalities such as hydrocephalus, as well as liver enlargement, jaundice, and eye problems such as chorioretinitis.  

Hemolytic disease of the newborn (B) is a condition that can occur when there is an incompatibility between the blood types of the mother and the fetus, but would not typically present with the other symptoms described.   Sepsis (C) is a potentially life-threatening condition caused by an infection, but would typically present with symptoms such as fever, rapid breathing, and decreased urine output.  

Congenital heart defect (D) is a broad category that can include a range of structural abnormalities of the heart, but would typically present with more specific symptoms depending on the type of defect.   Congenital hepatitis (E) is a rare condition that can occur when a newborn is infected with a virus such as hepatitis B or C, but would typically present with symptoms such as jaundice and liver enlargement, without the other symptoms described.  

Therefore, the most likely diagnosis for the newborn’s condition is A. Congenital toxoplasmosis. Confirmation of the diagnosis may require additional testing, such as serological testing for Toxoplasma gondii antibodies, and treatment may involve antiparasitic medication and supportive care for associated symptoms.


20. A woman at 12 weeks of her pregnancy developed a pain in her lower abdomen and mild bloody Discharge, which occurred with no apparent reason. Vaginal examination shows well-formed cervix with closed external orifice. The body of the uterus is enlarged according to her term of pregnancy. Make the right diagnosis:

A. Threatened miscarriage

B. Anembryonic pregnancy

C. Ongoing miscarriage

D. Imminent miscarriage

E. Incomplete miscarriage


Answer: Threatened miscarriage

Explanation

The most likely diagnosis for the woman at 12 weeks of her pregnancy who developed a pain in her lower abdomen and mild bloody discharge with a well-formed cervix with closed external orifice and an enlarged body of the uterus according to her term of pregnancy is A. Threatened miscarriage.   Threatened miscarriage is a common complication of early pregnancy characterized by vaginal bleeding and/or cramping, but with a closed cervix and no obvious signs of miscarriage on examination. The bleeding may be light or heavy and may be accompanied by abdominal pain or cramping.  

Anembryonic pregnancy (B) occurs when a fertilized egg implants in the uterus but does not develop into an embryo. This would typically be associated with an empty gestational sac on ultrasound.   Ongoing miscarriage (C) occurs when the pregnancy is not viable and is in the process of being lost. This would typically be associated with a dilated cervix and passage of fetal tissue.  

Imminent miscarriage (D) refers to a miscarriage that is likely to occur in the near future. This would typically be associated with cervical dilation and possibly the passage of fetal tissue.   Incomplete miscarriage (E) occurs when only part of the fetal tissue is passed, leaving some tissue in the uterus. This would typically be associated with bleeding, cramping, and possibly an open cervix.  

Therefore, the most likely diagnosis for the woman’s condition is A. Threatened miscarriage. Close monitoring and follow-up care may be necessary to determine the outcome of the pregnancy and to manage any associated complications.


21. In the demographic structure of a region, the people aged from 0 to 14 make up 31% of the population, while the 50+ age group makes up 20% of the population. What population structure characterizes such demographic situation most accurately?

A. Expansive population

B. Stationary population

C. Constrictive population

D. Emigration of the population

E. Migration of the population


Answer: Expansive population

Explanation

The demographic situation described, where the population aged from 0 to 14 makes up 31% of the population, while the 50+ age group makes up 20% of the population, is most accurately characterized as an expansive population.   An expansive population structure is characterized by a high proportion of young people (under 15 years old) and a relatively low proportion of elderly people (over 50 years old).

This type of population structure is typically associated with high fertility rates and low mortality rates, which can lead to population growth and a larger workforce.   A stationary population (B) is characterized by stable population growth, with a balance between the number of births and deaths.   A constrictive population (C) is characterized by a low proportion of young people and a high proportion of elderly people.

This type of population structure is typically associated with low fertility rates and high mortality rates, which can lead to population decline and a shrinking workforce.   Emigration (D) and migration (E) refer to the movement of people from one place to another, and are not directly related to population structure.   Therefore, the most accurate characterization of the demographic situation described is an expansive population structure.


22. A 13-year-old boy is complaining of a dry cough and shortness of breath. The onset of the disease was one year ago. He has brief asphyxia attacks that occur 1–2 times per month. Objectively, the child is anxious and has expiratory dyspnea, his skin is pale, his nasolabial triangle is cyanotic. His respiratory rate is 48/min. Percussion produces a banbox resonance over the lungs; auscultation detects weakened breathing with bilateral dry wheezing. Forced expiratory volume is 80% of the normal. What medicine should be prescribed to this boy?

A. Salbutamol

B. Euphyllin (Aminophylline)

C. Prednisolone

D. Indomethacin

E. Suprastin (Chloropyramine)


Answer: Salbutamol

Explanation

 


23. A patient with rheumatism presents with diastolic tremor of the chest wall («cat’s purr»), heart sound I that is amplified at the apex, diastolic murmur with presystolic amplification, a mitral opening sound, and an accent of the II 922521I1 4 heart sound over the pulmonary artery. What heart defect was observed in this patient?

A. Stenosis of the left atrioventricular orifice

B. Mitral valve insufficiency

C. Patent ductus arteriosus

D. Pulmonary artery stenosis

E. Aortic valve insufficiency


Answer: Stenosis of the left atrioventricular orifice

Explanation

The heart sounds and murmurs described in the patient with rheumatism, including diastolic tremor of the chest wall (“cat’s purr”), heart sound I that is amplified at the apex, diastolic murmur with presystolic amplification, a mitral opening sound, and an accent of the II heart sound over the pulmonary artery, suggest the presence of stenosis of the left atrioventricular orifice, also known as mitral stenosis.  

Mitral stenosis is a heart valve disorder that occurs when the mitral valve, which regulates blood flow between the left atrium and the left ventricle, becomes narrowed or obstructed. This can lead to a range of symptoms, including fatigue, shortness of breath, chest pain, and palpitations, as well as the characteristic diastolic murmur and other auscultatory findings described in this patient.  

Mitral valve insufficiency (B) occurs when the mitral valve does not close properly, leading to backflow of blood from the left ventricle into the left atrium. This would typically be associated with a systolic murmur and other auscultatory findings.   Patent ductus arteriosus (C) is a congenital heart defect that occurs when a blood vessel called the ductus arteriosus, which connects the pulmonary artery and the aorta during fetal development, fails to close after birth. This would typically be associated with a continuous murmur and other auscultatory findings.  

Pulmonary artery stenosis (D) is a heart valve disorder that occurs when the pulmonary valve, which controls blood flow between the right ventricle and the pulmonary artery, becomes narrowed or obstructed. This would typically be associated with a systolic murmur and other auscultatory findings.  

Aortic valve insufficiency (E) occurs when the aortic valve does not close properly, leading to backflow of blood from the aorta into the left ventricle. This would typically be associated with a diastolic murmur and other auscultatory findings.   Therefore, based on the symptoms and auscultatory findings described, the most likely diagnosis for this patient is A. Stenosis of the left atrioventricular orifice, or mitral stenosis.


24. At a factory with harmful working conditions, a complex of measures was being taken to reduce the morbidity among the workers. For better effect, the factory doctor was separately monitoring a group of people, who fall ill frequently and for a long time. What type of morbidity requires distinguishing such a group of people?

A. Morbidity with temporary disability

B. Morbidity associated with acute infections

C. General morbidity

D. Morbidity associated with major noncommunicable diseases

E. Hospitalized morbidity


Answer: Morbidity with temporary disability

Explanation

The type of morbidity that requires distinguishing a group of people who fall ill frequently and for a long time, and that the factory doctor is monitoring separately in order to improve the effectiveness of measures to reduce morbidity, is morbidity with temporary disability.   Morbidity with temporary disability refers to the number of cases of illness or injury that result in temporary absence from work or other daily activities.

This type of morbidity can be an important indicator of the impact of working conditions on the health of workers, as well as the effectiveness of measures taken to reduce morbidity and improve working conditions.   Acute infections (B) are a type of illness that typically have a sudden onset and short duration, such as colds or flu.  

General morbidity (C) refers to the overall level of illness or disease in a population, without distinguishing between different types of illness or severity of illness.   Morbidity associated with major noncommunicable diseases (D) refers to the prevalence of chronic diseases such as cardiovascular disease, cancer, diabetes, and chronic respiratory disease.  

Hospitalized morbidity (E) refers to the number of cases of illness or injury that require hospitalization.   Therefore, the type of morbidity that requires distinguishing a group of people who fall ill frequently and for a long time, and that the factory doctor is monitoring separately in order to improve the effectiveness of measures to reduce morbidity, is morbidity with temporary disability.


25. A 38-year-old patient has episodes of paroxysmal hypertension that reaches 240/120 mm Hg and is accompanied by nausea, vomiting, tachycardia, and excessive sweating. Hyperglycemia is observed in the blood during such episodes. After the episode, profuse urination occurs. Renal sonography has shown a new formation adjacent to the upper pole of the right kidney that might belong to the adrenal gland. What laboratory test will help clarify the diagnosis?

A. Urinary excretion of catecholamines and vanillylmandelic acid

B. Blood levels of thyroxine and thyroidstimulating hormone

C. Blood renin levels

D. Blood levels of insulin and C-peptide

E. Glomerular filtration rate, measured using the endogenous creatinine clearance rate


Answer: Urinary excretion of catecholamines and vanillylmandelic acid

Explanation

The symptoms described in the patient, including paroxysmal hypertension, nausea, vomiting, tachycardia, excessive sweating, hyperglycemia, and profuse urination, suggest the possibility of a pheochromocytoma, which is a rare tumor of the adrenal gland that produces excessive amounts of catecholamines. The presence of a new formation adjacent to the upper pole of the right kidney seen on renal sonography also supports this possibility. Therefore, the laboratory test that will help clarify the diagnosis is urinary excretion of catecholamines and vanillylmandelic acid, which can detect the presence of excess catecholamines in the urine.  

Urinary excretion of catecholamines and vanillylmandelic acid (A) is a useful diagnostic test for pheochromocytoma, as these substances are produced by the tumor and can be detected in the urine.   Blood levels of thyroxine and thyroid-stimulating hormone (B) are tests used to evaluate thyroid function and are not directly related to the diagnosis of pheochromocytoma.   Blood renin levels (C) are a test used to evaluate the renin-angiotensin-aldosterone system and are not directly related to the diagnosis of pheochromocytoma.  

Blood levels of insulin and C-peptide (D) are tests used to evaluate insulin secretion and are not directly related to the diagnosis of pheochromocytoma.   Glomerular filtration rate measured using the endogenous creatinine clearance rate (E) is a test used to evaluate kidney function and is not directly related to the diagnosis of pheochromocytoma.   Therefore, the correct answer is A. Urinary excretion of catecholamines and vanillylmandelic acid, as it is the most appropriate laboratory test to help clarify the diagnosis of pheochromocytoma.


26. During physical exertion, a man with a cerebral artery aneurysm suddenly developed severe headache and vomiting and fainted. The patient is agitated and wants to run somewhere. His pulse is 62/min., rhythmic, blood pressure – 140/90 mm Hg, the body temperature is 37.5oC. He has nuchal rigidity and positive Kernig’s sign. Focal neurological signs are absent. Make the provisional diagnosis:

A. Subarachnoid hemorrhage

B. Hypertensive crisis

C. Ischemic stroke

D. Meningitis

E. Brain hemorrhage


Answer: Subarachnoid hemorrhage

Explanation

The sudden onset of severe headache, vomiting, and loss of consciousness in a patient with a cerebral artery aneurysm, along with nuchal rigidity and a positive Kernig’s sign, suggest the possibility of a subarachnoid hemorrhage. The patient’s agitated state and desire to run somewhere may also be indicative of a neurological emergency.   A subarachnoid hemorrhage (A) occurs when there is bleeding in the space between the brain and the thin tissues that cover the brain. It is a medical emergency that can be life-threatening if not treated promptly.  

Hypertensive crisis (B) is a sudden and severe increase in blood pressure that can cause symptoms such as headache, nausea, vomiting, and confusion. However, the presence of a cerebral artery aneurysm and the patient’s other symptoms make a subarachnoid hemorrhage a more likely diagnosis.  

Ischemic stroke (C) occurs when blood flow to the brain is blocked, usually by a blood clot. This can cause symptoms such as weakness, numbness, and difficulty speaking. The sudden onset of severe headache and vomiting, along with the presence of nuchal rigidity and a positive Kernig’s sign, make a subarachnoid hemorrhage a more likely diagnosis.   Meningitis (D) is an inflammation of the membranes that surround the brain and spinal cord. It can cause symptoms such as headache, fever, and stiff neck.

However, the sudden onset of severe headache, vomiting, loss of consciousness, and positive Kernig’s sign, along with the presence of a cerebral artery aneurysm, make a subarachnoid hemorrhage a more likely diagnosis.   Brain hemorrhage (E) is a type of bleeding that occurs within the brain tissue itself. It can cause symptoms such as headache, nausea, vomiting, and loss of consciousness.

However, the presence of a cerebral artery aneurysm, along with the sudden onset of severe headache, vomiting, and positive Kernig’s sign, make a subarachnoid hemorrhage a more likely diagnosis.   Therefore, based on the symptoms described, the provisional diagnosis is A. Subarachnoid hemorrhage. Prompt medical attention and treatment are necessary in order to manage this condition and prevent further complications.


27. A 14-year-old boy developed a sympatho-adrenal crisis. What medicine was indicated for crisis relief in this case?

A. Propranolol

B. Digoxin

C. Thiopental sodium

D. Euphyllin (Aminophylline)

E. Drotaverine


Answer: Propranolol

Explanation

A sympatho-adrenal crisis is a condition characterized by the excessive release of adrenaline and noradrenaline, which can result in symptoms such as high blood pressure, tachycardia, sweating, and anxiety. The appropriate medicine for crisis relief in this case is propranolol.  

Propranolol (A) is a beta-blocker that blocks the effects of adrenaline and noradrenaline on the body, reducing blood pressure, heart rate, and other symptoms of the sympatho-adrenal crisis.   Digoxin (B) is a medication used to treat heart failure and certain arrhythmias, but it is not indicated for the treatment of a sympatho-adrenal crisis.  

Thiopental sodium (C) is a short-acting barbiturate that is used as an anesthetic agent, but it is not indicated for the treatment of a sympatho-adrenal crisis.   Euphyllin (Aminophylline) (D) is a medication used to treat asthma and other respiratory conditions, but it is not indicated for the treatment of a sympatho-adrenal crisis.  

Drotaverine (E) is a medication used to treat smooth muscle spasms, but it is not indicated for the treatment of a sympatho-adrenal crisis.   Therefore, the correct answer is A. Propranolol, as it is the appropriate medication for crisis relief in a sympatho-adrenal crisis, as it blocks the effects of adrenaline and noradrenaline on the body.

However, it is important to note that the treatment of a sympatho-adrenal crisis should be performed in a medical setting by a healthcare professional, as this is a potentially life-threatening condition that requires prompt and appropriate treatment.


28. In the morning, an 8-year-old child started complaining of general weakness and pain during swallowing. At noon, her parents called for a doctor, because the girl’s weakness was progressing and she developed a bilateral swelling slightly below and in front of her ears. Make the right diagnosis:

A. Mumps

B. Lymphadenopathy

C. Tumor of the salivary glands

D. Dermoid cysts

E. Tumor of the carotid body


Answer: Mumps

Explanation

The symptoms described in the child, including general weakness, pain during swallowing, and bilateral swelling below and in front of the ears, suggest the possibility of mumps. Therefore, the correct diagnosis is A. Mumps.   Mumps (A) is a viral infection that primarily affects the salivary glands, causing swelling and tenderness in one or both of the parotid glands, which are located below and in front of the ears. Other symptoms may include general weakness, fever, headache, muscle aches, and pain during swallowing.  

Lymphadenopathy (B) refers to the enlargement of lymph nodes, which can be caused by a variety of conditions such as infection, inflammation, or cancer. However, the presence of swelling in the parotid gland region, along with the other symptoms described, makes mumps a more likely diagnosis.  

A tumor of the salivary glands (C) is a rare condition that can cause swelling and tenderness in the parotid gland region, but it is less common in children and is not typically associated with the other symptoms described.   Dermoid cysts (D) are benign growths that can occur anywhere in the body, including the head and neck area. However, they do not typically cause the symptoms described in this case.  

A tumor of the carotid body (E) is a rare type of tumor that arises from the carotid body, which is a small structure located near the carotid artery in the neck. This type of tumor is not typically associated with the symptoms described in this case.   Therefore, based on the symptoms described, the correct diagnosis is A. Mumps. Treatment typically involves supportive care such as rest, pain relief, and hydration, as well as isolation to prevent the spread of the virus to others. Vaccination is the most effective way to prevent mumps.


29. A 58-year-old patient, a heavy drinker and smoker, came to a hospital with complaints of constant coughing and shortness of breath. Lately, he has been losing weight. His cervical lymph nodes are enlarged and dense, the tissues above them exhibit no tension. Chest X-ray has shown fibrosis of an upper pulmonary lobe and left-sided pleurisy. The pleural fluid is straw-colored, with protein levels of 52 g/L and a high lymphocyte count. Malignant cells were not detected. Inoculation of the pleural fluid produced no microbial growth one week later. Make the right diagnosis:

A. Pulmonary tuberculosis

B. Systemic lupus erythematosus

C. Sarcoidosis

D. Bronchiectasis

E. Atypical pneumonia


Answer: Pulmonary tuberculosis

Explanation

The symptoms and findings described in the patient, including constant coughing, shortness of breath, weight loss, enlarged cervical lymph nodes, fibrosis of an upper pulmonary lobe, left-sided pleurisy, and straw-colored pleural fluid with a high lymphocyte count, suggest the possibility of pulmonary tuberculosis. Therefore, the correct diagnosis is A.

Pulmonary tuberculosis.   Pulmonary tuberculosis (A) is a bacterial infection caused by Mycobacterium tuberculosis that primarily affects the lungs. It can cause symptoms such as coughing, shortness of breath, fever, night sweats, and weight loss. Enlarged cervical lymph nodes, fibrosis of an upper pulmonary lobe, and pleurisy are common findings in patients with pulmonary tuberculosis.

The straw-colored pleural fluid with a high lymphocyte count is also consistent with tuberculosis.   Systemic lupus erythematosus (B) is an autoimmune disease that can affect multiple organs and systems in the body, but it is not typically associated with the symptoms and findings described in this case.  

Sarcoidosis (C) is a condition characterized by the formation of granulomas in various organs, including the lungs, lymph nodes, and skin. It can cause symptoms such as coughing, shortness of breath, and weight loss, but the pleural fluid findings described in this case are not typical of sarcoidosis.  

Bronchiectasis (D) is a condition characterized by the abnormal widening of airways in the lungs, which can lead to symptoms such as coughing, shortness of breath, and recurrent lung infections. However, the cervical lymph node enlargement, fibrosis of an upper pulmonary lobe, and pleural fluid findings described in this case are not typical of bronchiectasis.  

Atypical pneumonia (E) is a type of pneumonia caused by bacteria other than Streptococcus pneumoniae, which is the most common cause of bacterial pneumonia. The symptoms and findings described in this case are not typical of atypical pneumonia.   Therefore, based on the symptoms and findings described, the correct diagnosis is A. Pulmonary tuberculosis. Treatment typically involves a combination of antibiotics for several months, as well as supportive care for any related symptoms.


30. A laboratory has received a milk sample for analysis. The findings are as follows: color is whitish, smell is normal, taste is normal for milk, specific gravity is 1,038, acidity is 35oT (degrees Turner), fats is 3.2%. What is the milk quality? 922521I1 5

A. The milk is of poor quality

B. The milk is of reduced quality

C. The milk is conditionally fit for use

D. The milk is a counterfeit product

E. The milk is of good quality


Answer:  The milk is of poor quality

Explanation

The findings from the milk sample analysis suggest that the milk is of poor quality. The specific gravity of 1.038 is higher than the normal range for milk, which suggests possible adulteration with water or other substances. The acidity level of 35oT is also higher than the normal range for fresh milk, indicating that the milk may be starting to spoil.

The fat content of 3.2% is within the normal range for milk, but it does not necessarily indicate good quality on its own.   Therefore, the correct answer is A. The milk is of poor quality. This milk should not be consumed and should be discarded to prevent any potential health risks. It is important to ensure that milk and other dairy products are obtained from reputable sources and are properly stored and handled to ensure their quality and safety for consumption.


31. A 7-year-old child fell ill 2 weeks ago, when he developed a runny nose. An otolaryngologist was consulted about sanguinopurulent discharge from the child’s nose and maceration of his wings of the nose and upper lip. Rhinoscopy has detected whitish-gray foci on the nasal septum. The oropharyngeal mucosa is without changes. Make the right diagnosis:

A. Nasal diphtheria

B. Adenovirus infection

C. Maxillary sinusitis

D. Rhinovirus infection

E. Allergic rhinitis


Answer: Nasal diphtheria

Explanation

The symptoms and findings described in the child, including sanguinopurulent discharge from the nose, maceration of the wings of the nose and upper lip, and whitish-gray foci on the nasal septum, suggest the possibility of nasal diphtheria. Therefore, the correct diagnosis is A. Nasal diphtheria.   Nasal diphtheria (A) is a bacterial infection caused by Corynebacterium diphtheriae that primarily affects the nose and throat. It can cause symptoms such as runny nose, bloody or purulent nasal discharge, and maceration of the skin around the nose and mouth.

The whitish-gray foci on the nasal septum are characteristic of nasal diphtheria.   Adenovirus infection (B) is a viral infection that can cause symptoms such as runny nose, cough, and fever, but it is not typically associated with the nasal discharge and nasal septum findings described in this case.   Maxillary sinusitis (C) is an inflammation or infection of the maxillary sinuses, which can cause symptoms such as facial pain, headache, and nasal discharge, but it does not typically present with the specific findings described in this case.  

Rhinovirus infection (D) is a viral infection that can cause symptoms such as runny nose, nasal congestion, and sore throat, but it does not typically present with the specific findings described in this case.   Allergic rhinitis (E) is a condition that can cause symptoms such as runny nose, nasal congestion, and sneezing, but it does not typically present with the specific findings described in this case.  

Therefore, based on the symptoms and findings described, the correct diagnosis is A. Nasal diphtheria. Treatment typically involves antibiotics and antitoxin therapy, as well as supportive care for any related symptoms. It is important to isolate the patient and take appropriate infection control measures to prevent the spread of the infection to others.


32. An 18-year-old young man complains of pain in his knee and ankle joints and a fever of 39.5oC. A week and a half before, he had a case of respiratory disease. His body temperature is 38.5oC, his knee and ankle joints are swollen. His pulse is 106/min., rhythmic. His blood pressure is 90/60 mm Hg. The heart borders are normal, the heart sounds are weakened, and there is a soft systolic murmur at the apex. What parameter was most closely associated with the possible etiology of this process?

A. Antistreptolysin 0

B. Seromucoid

C. Creatine kinase

D. Rheumatoid factor

E. α1-antitrypsin


Answer: Antistreptolysin 0

Explanation

The symptoms described in the young man, including joint pain, fever, joint swelling, and a soft systolic murmur at the apex of the heart, suggest the possibility of rheumatic fever. Therefore, the parameter most closely associated with the possible etiology of this process is A. Antistreptolysin 0.   Rheumatic fever is a complication of untreated or inadequately treated streptococcal infections, particularly group A beta-hemolytic streptococcus. It can cause symptoms such as joint pain, fever, and a characteristic rash, as well as cardiac complications such as a soft systolic murmur at the apex of the heart.

Antistreptolysin 0 (ASO) is an antibody produced in response to a streptococcal infection, and elevated levels of ASO are a common finding in patients with rheumatic fever.   Seromucoid (B) is a type of acute phase reactant that can be elevated in response to inflammation or tissue damage, but it is not specific to rheumatic fever.  

Creatine kinase (C) is an enzyme found in muscle tissue that can be elevated in response to muscle damage or injury, but it is not specific to rheumatic fever.   Rheumatoid factor (D) is an antibody that can be elevated in patients with rheumatoid arthritis, but it is not typically associated with rheumatic fever.  

α1-antitrypsin (E) is a protein that plays a role in regulating inflammation, but it is not typically associated with rheumatic fever.   Therefore, based on the symptoms described, the parameter most closely associated with the possible etiology of this process is A. Antistreptolysin 0. Additional tests and evaluations may be needed to confirm the diagnosis of rheumatic fever and determine the appropriate treatment.


33. A postparturient woman came to a doctor on the 14th day after giving birth. She complained of a sudden pain, hyperemia, and induration in her left breast, fever of 39oC, headache, and indisposition. There is a fissure in the nipple, the left breast is enlarged, during its palpation the pain intensifies. What pathology is suspected in this case?

A. Lactational mastitis

B. Breast cancer

C. Suppurated cyst of the left breast

D. Phlegmon of the breast

E. Fibroadenoma of the left breast


Answer: Lactational mastitis

Explanation

The symptoms and findings described in the postparturient woman, including sudden pain, hyperemia, induration, fever, headache, indisposition, fissure in the nipple, and pain on palpation of the left breast, suggest the possibility of lactational mastitis. Therefore, the correct diagnosis is A. Lactational mastitis.   Lactational mastitis (A) is a bacterial infection that can occur in breastfeeding women, typically within the first few weeks after giving birth. It can cause symptoms such as breast pain, redness, swelling, fever, and flu-like symptoms.

The fissure in the nipple may allow bacteria to enter the breast tissue and contribute to the development of mastitis.   Breast cancer (B) is a malignant tumor that can occur in the breast tissue, but it typically does not present with the acute onset of symptoms described in this case.   Suppurated cyst of the left breast (C) is a collection of pus within a cyst in the breast tissue, but it does not typically present with the acute onset of symptoms described in this case.  

Phlegmon of the breast (D) is a diffuse bacterial infection of the breast tissue that can cause symptoms such as breast pain, redness, swelling, and fever. However, it is typically not specific to lactation and breastfeeding.   Fibroadenoma of the left breast (E) is a benign tumor that can occur in the breast tissue, but it does not typically present with the acute onset of symptoms described in this case.  

Therefore, based on the symptoms and findings described, the correct diagnosis is A. Lactational mastitis. Treatment typically involves antibiotics, pain relief, and supportive care for breastfeeding, such as ensuring proper latch and drainage of the affected breast. It is important for the patient to continue breastfeeding to prevent further complications and promote healing.


34. A 15-year-old patient presents with delayed physical development and periodically develops icteric skin. Spleen is 16x12x10 cm, cholecystolithiasis is observed in the patient, there is a skin ulcer on the left calf in its lower third. The blood has: erythrocytes – 3.0 · 1012/L, Hb – 90 g/L, color index – 1.0, microspherocytosis, reticulocytosis. Total serum bilirubin – 56 mcmol/L, indirect bilirubin – 38 mcmol/L. What treatment method is optimal in this case?

A. Splenectomy

B. Omentohepatopexy

C. Spleen transplant

D. Omentosplenopexy

E. Portocaval anastomosis


Answer: Splenectomy

Explanation

The symptoms and findings described in the 15-year-old patient, including delayed physical development, icteric skin, enlarged spleen, cholecystolithiasis, skin ulcer, microspherocytosis, reticulocytosis, and elevated bilirubin levels, suggest the possibility of hereditary spherocytosis. Therefore, the optimal treatment method in this case is A. Splenectomy.   Hereditary spherocytosis is a genetic disorder that affects the red blood cells and can cause symptoms such as anemia, jaundice, and an enlarged spleen.

Splenectomy is the optimal treatment method for this condition, as it can alleviate symptoms and reduce the risk of complications such as gallstones and leg ulcers.   Omentohepatopexy (B) is a surgical procedure that involves attaching the omentum (a fatty tissue in the abdomen) to the liver, but it is not typically used to treat hereditary spherocytosis.  

Spleen transplant (C) is a potential treatment option for some patients with hereditary spherocytosis, but it is not typically the first-line treatment and is only considered in certain cases.   Omentosplenopexy (D) is a surgical procedure that involves attaching the omentum to the spleen, but it is not typically used to treat hereditary spherocytosis.  

Portocaval anastomosis (E) is a surgical procedure that involves creating a connection between the portal vein and the vena cava, but it is not typically used to treat hereditary spherocytosis.   Therefore, based on the symptoms and findings described, the optimal treatment method in this case is A. Splenectomy. It is important for the patient to continue to be monitored and managed appropriately for any related symptoms or complications following the procedure.


35. A 22-year-old woman at 11–12 weeks of pregnancy came to a maternity clinic. Examination has shown a positive Wasserman reaction. A dermatologist diagnosed her with secondary latent syphilis. What are the tactics of pregnancy management in such case?

A. Artificial termination of the pregnancy after a course of antisyphilitic therapy

B. Prolongation of the pregnancy after a course of antisyphilitic therapy

C. Artificial termination of the pregnancy after the diagnosis is made

D. Artificial termination of the pregnancy after normalization of the patient’s Wasserman reaction

E. Three antisyphilitic treatment courses throughout the pregnancy


Answer: Artificial termination of the pregnancy after a course of antisyphilitic therapy

Explanation

The diagnosis of secondary latent syphilis in a pregnant woman is a serious concern, as untreated or inadequately treated syphilis can pose significant risks to both the mother and the developing fetus. Therefore, the optimal tactic for pregnancy management in this case is A. Artificial termination of the pregnancy after a course of antisyphilitic therapy.  

Syphilis is a sexually transmitted infection caused by the bacterium Treponema pallidum. It can be transmitted from mother to fetus during pregnancy and can cause serious complications such as stillbirth, neonatal death, and long-term neurological and developmental problems in surviving infants. Treatment with appropriate antibiotics is necessary to prevent these complications and to treat the infection in the mother.  

In this case, the optimal tactic for pregnancy management is to initiate a course of antisyphilitic therapy as soon as possible to treat the infection in the mother. Once the mother has completed a course of treatment and the infection has been adequately treated, artificial termination of the pregnancy may be considered to prevent potential complications in the developing fetus. This decision should be made in consultation with the patient and based on individual circumstances and medical history.  

Prolongation of the pregnancy (B) without treatment is not recommended, as it poses significant risks to the developing fetus.   Artificial termination of the pregnancy after the diagnosis is made (C) without treatment is not recommended, as it does not address the underlying infection in the mother and may not prevent potential complications in the developing fetus.  

Artificial termination of the pregnancy after normalization of the patient’s Wasserman reaction (D) without treatment is not recommended, as it does not address the underlying infection in the mother and may not prevent potential complications in the developing fetus.   Three antisyphilitic treatment courses throughout the pregnancy (E) may be considered in some cases of syphilis during pregnancy, but the optimal tactic in this case would be to initiate treatment as soon as possible and consider artificial termination of the pregnancy after the infection has been adequately treated.


36. A lumbar puncture was made for a newborn with a suspected intracranial birth injury. Bloody cerebrospinal fluid was obtained. What type of hemorrhage was observed in this case?

A. Subarachnoid hemorrhage

B. Cephalohematoma

C. Subtentorial hemorrhage

D. Epidural hemorrhage

E. Supratentorial hemorrhage


Answer: Subarachnoid hemorrhage

Explanation

The presence of bloody cerebrospinal fluid obtained through lumbar puncture in a newborn with suspected intracranial birth injury suggests the possibility of a subarachnoid hemorrhage. Therefore, the correct answer is A. Subarachnoid hemorrhage.   Subarachnoid hemorrhage (SAH) is a type of bleeding that occurs in the space between the arachnoid membrane and the pia mater, which are the layers that surround the brain and spinal cord. SAH can be caused by a variety of factors, including trauma, aneurysms, and bleeding disorders.

In newborns, SAH is most commonly associated with birth injury and can be a significant cause of morbidity and mortality.   Cephalohematoma (B) is a collection of blood that occurs between the skull and the periosteum, typically as a result of birth trauma. It is not typically associated with bloody cerebrospinal fluid obtained through lumbar puncture.  

Subtentorial hemorrhage (C) is a type of bleeding that occurs below the tentorium, which is a membrane that separates the cerebrum from the cerebellum. It is not typically associated with bloody cerebrospinal fluid obtained through lumbar puncture.   Epidural hemorrhage (D) is a type of bleeding that occurs between the skull and the dura mater, typically as a result of trauma.

It is not typically associated with bloody cerebrospinal fluid obtained through lumbar puncture.   Supratentorial hemorrhage (E) is a type of bleeding that occurs above the tentorium, typically as a result of trauma or vascular malformations. It is not typically associated with bloody cerebrospinal fluid obtained through lumbar puncture.   Therefore, based on the symptoms and findings described, the correct answer is A. Subarachnoid hemorrhage. Further evaluation and management may be needed to determine the underlying cause and optimal treatment approach for the newborn.


37. The mother of a newborn child is suffering from chronic pyelonephritis and had a case of acute respiratory viral infection before giving birth. The delivery is full-term, the waters did not break for a long time. On the 2nd day of life the baby developed an erythematous rash. Blisters of about 1 cm in size appeared. They are filled with seropurulent contents and their lancing results in formation of erosions. The Nikolsky’s sign is positive. The baby is inert and has a subfebrile body temperature. Make the diagnosis: 922521I1 6

A. Neonatal pemphigus

B. Vesiculopustulosis

C. Sepsis

D. Ritter’s disease of the newborn

E. Pseudofurunculosis


Answer: Neonatal pemphigus

Explanation

The symptoms and findings described in the newborn, including the presence of an erythematous rash with blisters filled with seropurulent contents, positive Nikolsky’s sign, subfebrile body temperature, and maternal history of chronic pyelonephritis, suggest the possibility of neonatal pemphigus. Therefore, the correct diagnosis is A.

Neonatal pemphigus.   Neonatal pemphigus is a rare autoimmune disorder that can occur in newborns born to mothers with autoimmune diseases, such as chronic pyelonephritis. It can cause symptoms such as skin blisters, erosions, and crusting, as well as systemic symptoms such as fever and lethargy. The condition is caused by the transfer of maternal autoantibodies to the newborn, which attack the newborn’s skin cells.   Vesiculopustulosis (B) is a term used to describe a group of skin conditions that can cause vesicles and pustules in newborns, but it does not typically present with the positive Nikolsky’s sign or seropurulent contents described in this case.  

Sepsis (C) is a potentially life-threatening condition that can occur in newborns and is characterized by the presence of bacteria or other microorganisms in the blood. While sepsis can cause skin changes, it typically does not present with the specific skin findings described in this case.   Ritter’s disease of the newborn (D), also known as staphylococcal scalded skin syndrome, is a bacterial infection that can cause skin blistering and peeling in newborns, but it typically presents with more widespread skin involvement than the localized blisters described in this case.  

Pseudofurunculosis (E) is a skin condition that can cause pustules and papules on the skin, but it typically does not present with the specific skin findings described in this case.   Therefore, based on the symptoms and findings described, the correct diagnosis is A. Neonatal pemphigus. Treatment typically involves the use of systemic corticosteroids and immunosuppressive agents, as well as supportive care for the newborn’s skin and overall health. It is important to manage the condition promptly to prevent potential complications and improve outcomes.


38. A 34-year-old patient has been suffering from pulmonary tuberculosis for the last 7 years. He complains of muscle weakness, weight loss, diarrhea, and frequent urination. His skin, gums, and buccal mucosa are hyperpigmented. His blood pressure is 90/58 mm Hg. In the blood: erythrocytes – 3.1 · 1012/L, Hb – 95 g/L, color index – 0.92; leukocytes – 9.4 · 109/L, eosinophils – 7%, segmented neutrophils – 45%, band neutrophils – 1%, lymphocytes – 40%, monocytes – 7%, Na+ – 115 mmol/L, + – 7.3 mmol/L. Make the diagnosis:

A. Primary insufficiency of the adrenal cortex

B. Primary hyperaldosteronism

C. Congenital hyperplasia of the adrenal cortex

D. Pheochromocytoma

E. Diabetes insipidus


Answer: Primary insufficiency of the adrenal cortex

Explanation

The symptoms and findings described in the patient, including muscle weakness, weight loss, hyperpigmentation of the skin, gums, and buccal mucosa, and electrolyte imbalances (low sodium, high potassium) in the blood, suggest the possibility of primary insufficiency of the adrenal cortex, also known as Addison’s disease. Therefore, the correct diagnosis is A.

Primary insufficiency of the adrenal cortex.   Addison’s disease is a rare condition in which the adrenal glands do not produce enough cortisol and aldosterone, two hormones that are essential for regulating metabolism and electrolyte balance in the body. The condition can cause a wide range of symptoms, including fatigue, weight loss, muscle weakness, hyperpigmentation, and electrolyte imbalances. In some cases, it can also cause gastrointestinal symptoms such as diarrhea and frequent urination.  

Primary hyperaldosteronism (B) is a condition in which the adrenal glands produce too much aldosterone, which can cause high blood pressure and electrolyte imbalances, but it typically does not present with the specific symptoms and findings described in this case.   Congenital hyperplasia of the adrenal cortex (C) is a genetic disorder that affects the adrenal glands and can cause a wide range of symptoms, including hyperpigmentation, electrolyte imbalances, and hormonal imbalances, but it typically presents in childhood and adolescence and may not explain the patient’s history of tuberculosis.  

Pheochromocytoma (D) is a rare tumor that can develop in the adrenal glands and produce excess adrenaline and noradrenaline, which can cause symptoms such as high blood pressure, sweating, and palpitations, but it typically does not explain the specific symptoms and findings described in this case.   Diabetes insipidus (E) is a condition in which the kidneys are unable to conserve water, which can cause excessive thirst and frequent urination, but it typically does not explain the other symptoms and findings described in this case.  

Therefore, based on the symptoms and findings described, the correct diagnosis is A. Primary insufficiency of the adrenal cortex. Treatment typically involves the use of hormone replacement therapy to replace the deficient hormones and manage symptoms and complications. It is important to manage the condition promptly to prevent potential complications and improve outcomes.


39. A 35-year-old woman has red swollen areas on the dorsal surface of her hands after a severe nervous strain, which was followed by formation of small inflamed nodules, vesicles, and later erosions with significant discharge of a serous fluid. This process was accompanied by severe itching. Make the right diagnosis:

A. True eczema

B. Microbial eczema

C. Toxicoderma

D. Simple contact dermatitis

E. Allergic dermatitis


Answer: True eczema

Explanation

The symptoms and findings described in the patient, including red swollen areas on the dorsal surface of the hands after a severe nervous strain, formation of small inflamed nodules, vesicles, and later erosions with discharge of serous fluid, and severe itching, suggest the possibility of true eczema. Therefore, the correct diagnosis is A. True eczema.  

True eczema, also known as atopic dermatitis, is a chronic inflammatory skin condition that is characterized by red, itchy, swollen, and flaky skin. It can be triggered by a variety of factors, including stress, environmental allergens, and irritants. The condition can cause significant discomfort, and in severe cases, it can lead to complications such as skin infections and sleep disturbances.   Microbial eczema (B) is a type of eczema that is caused by bacterial or fungal infection of the skin, and it typically presents with different symptoms and findings than those described in this case.  

Toxicoderma (C) is a term used to describe a group of skin conditions that are caused by exposure to toxic substances, such as chemicals or medications. While toxicoderma can cause skin changes, it typically presents with different symptoms and findings than those described in this case.   Simple contact dermatitis (D) is a type of eczema that is caused by direct contact with an irritant or allergen, and it typically presents with different symptoms and findings than those described in this case.  

Allergic dermatitis (E) is a type of eczema that is caused by an allergic reaction to a substance, such as a cosmetic or medication, and it typically presents with different symptoms and findings than those described in this case.   Therefore, based on the symptoms and findings described, the correct diagnosis is A. True eczema. Treatment typically involves the use of topical corticosteroids and other medications to relieve itching and inflammation, as well as avoiding triggers and practicing good skin care. It is important to manage the condition promptly to prevent potential complications and improve outcomes.


40. A 48-year-old patient, whose blood test has resulted in positive Wasseman and sedimentation reactions thrice, came to a doctor. His treponema pallidum immobilization test was positive as well. Examination has shown no clinical manifestations of syphilis in the patient’s skin, internal organs or nervous system. Make the right diagnosis:

A. Early latent syphilis

B. Latent recurrent syphilis

C. Tertiary syphilis

D. Seropositive primary syphilis

E. Recurrent secondary syphilis


Answer: Early latent syphilis

Explanation

The positive Wasseman and sedimentation reactions, as well as a positive treponema pallidum immobilization test, suggest the possibility of syphilis in the patient. However, the absence of clinical manifestations of syphilis in the skin, internal organs, or nervous system, suggests that the patient may be in the early latent stage of the disease.

Therefore, the correct diagnosis is A. Early latent syphilis.   Syphilis is a sexually transmitted infection caused by the bacterium Treponema pallidum. The disease progresses through several stages, including primary, secondary, latent, and tertiary syphilis. In the early latent stage, there are no clinical manifestations of the disease, but the patient tests positive for syphilis on blood tests.  


Latent recurrent syphilis (B) is a stage of the disease that occurs after the early latent stage, in which the patient experiences a recurrence of symptoms despite being asymptomatic previously.   Tertiary syphilis (C) is a stage of the disease that occurs after latent syphilis, in which the patient may develop serious complications such as neurosyphilis, cardiovascular syphilis, and gummatous syphilis.  

Seropositive primary syphilis (D) is a stage of the disease that occurs shortly after infection, characterized by the presence of a chancre, a painless sore on the genitals, anus, or mouth, and a positive blood test for syphilis.   Recurrent secondary syphilis (E) is a stage of the disease that occurs after primary syphilis, characterized by the presence of skin rash, mucous membrane lesions, and other symptoms.  

Therefore, based on the symptoms and findings described, the correct diagnosis is A. Early latent syphilis. Treatment typically involves the use of antibiotics, such as penicillin, to cure the infection and prevent potential complications. It is important to diagnose and manage the condition promptly to prevent further spread of the disease and improve outcomes.


41. A 50-year-old man was hospitalized with complaints of blood in the urine. There are no pain or urination disorders. Hematuria is observed for the last 3 days. The kidneys are not palpable, the suprapubic area is normal, the external genitalia have no pathology. Rectal examination detects no prostatic enlargement. Cystoscopy detects no changes. What disease is suspected first?

A. Kidney cancer

B. Renal dystopia

C. Necrotizing papillitis

D. Varicocele

E. Tuberculosis of the urinary bladder


Answer: Kidney cancer

Explanation

In a patient with hematuria, the possibility of kidney cancer should be considered. The absence of pain or urination disorders, normal findings on physical examination, including normal kidney palpation, suprapubic area, external genitalia, and normal cystoscopy suggests that the source of bleeding is likely within the kidneys.

Therefore, the correct answer is A. Kidney cancer.   Kidney cancer is a malignant tumor that originates in the cells of the kidney. The most common symptom of kidney cancer is hematuria, which may be visible or microscopic. Other symptoms may include abdominal or flank pain, weight loss, and fever. In some cases, kidney cancer may be asymptomatic and detected incidentally on imaging studies.  

Renal dystopia (B) is a congenital abnormality in which the kidney is located in an abnormal position within the abdomen, but it typically does not cause hematuria or other symptoms unless there is an associated condition or complication.   Necrotizing papillitis (C) is a condition in which the papilla, the small projections in the kidney where urine drains into the ureter, becomes inflamed and may die, causing pain, fever, and occasionally hematuria.   Varicocele (D) is a condition in which the veins in the scrotum become enlarged and twisted, causing discomfort and sometimes infertility, but it is not typically associated with hematuria.  

Tuberculosis of the urinary bladder (E) is a bacterial infection that can affect the bladder and cause symptoms such as dysuria and hematuria, but it is unlikely to cause isolated hematuria without other urinary symptoms.   Therefore, based on the symptoms and findings described, the correct diagnosis is A. Kidney cancer. Additional diagnostic tests, such as imaging studies and biopsy, may be necessary to confirm the diagnosis and determine the best course of treatment. It is important to manage the condition promptly to prevent potential complications and improve outcomes.


42. A 28-year-old patient complains of chills in his feet and fingers and pain in his legs during walking. The disease onset was one year ago. The skin of his legs is pale and cold; low turgor and hypotrichosis are observed. Pulsation over the femoral and popliteal arteries is weakened, on the pedal arteries it is palpable after a nitroglycerin test. Rheographic index is <1. Make the right diagnosis:

A. Obliterating endarteritis

B. Chronic thrombophlebitis

C. Wegener vasculitis

D. Arteriosclerosis obliterans

E. Raynaud syndrome


Answer: Obliterating endarteritis

Explanation

The symptoms and findings described in the patient, including chills in the feet and fingers, pain in the legs during walking, pale and cold skin of the legs, low turgor and hypotrichosis, weakened pulsation over the femoral and popliteal arteries, and a rheographic index of <1, suggest the possibility of obliterating endarteritis.

Therefore, the correct diagnosis is A. Obliterating endarteritis, also known as peripheral artery disease.   Obliterating endarteritis is a condition in which the arteries of the limbs become narrowed and blocked due to inflammation and the formation of plaques, leading to reduced blood flow and oxygen supply to the affected tissues.

The condition typically affects the arteries of the legs, causing symptoms such as pain, cramping, and weakness during walking or other activities.   Chronic thrombophlebitis (B) is a condition in which the veins become inflamed and develop blood clots, which can cause pain, swelling, and discoloration of the affected area, but it typically does not cause the specific symptoms and findings described in this case.  

Wegener vasculitis (C) is a rare condition in which the blood vessels become inflamed, leading to damage to various organs and tissues, but it typically does not cause the specific symptoms and findings described in this case.   Arteriosclerosis obliterans (D) is a condition in which the arteries become hardened and narrowed due to the buildup of plaque, leading to reduced blood flow and oxygen supply to the affected tissues.

It is similar to obliterating endarteritis, but it typically affects the larger arteries and is more commonly associated with cardiovascular disease.   Raynaud syndrome (E) is a condition in which the blood vessels in the fingers and toes become narrow and restrict blood flow in response to cold or stress, leading to symptoms such as pain, numbness, and tingling, but it typically does not cause the specific symptoms and findings described in this case.   Therefore, based on the symptoms and findings described, the correct diagnosis is A.

Obliterating endarteritis. Treatment typically involves the use of medications to improve blood flow, lifestyle modifications, and in severe cases, surgery or other interventions to restore blood flow to the affected area. It is important to manage the condition promptly to prevent potential complications and improve outcomes.


43. A 20-year-old woman complains of amenorrhea. She has hirsutism and obesity with predominant distribution of adipose tissue on the face, neck, and upper torso. On her face she has acne vulgaris, on her skin there are stretch marks. The blood pressure is 170/100 mm Hg. Her mental development and intelligence are not impaired. Gynecological status shows moderate hairiness of the external genitalia and acute hypoplasia of the vagina and uterus. Make the right diagnosis:

A. Cushing syndrome

B. Stein-Leventhal syndrome

C. Turner syndrome

D. Sheehan syndrome

E. Babinski-Froehlich syndrome


Answer: Cushing syndrome

Explanation

The symptoms and findings described in the patient, including amenorrhea, hirsutism, obesity with predominant distribution of adipose tissue on the face, neck, and upper torso, acne vulgaris, stretch marks, hypertension, and acute hypoplasia of the vagina and uterus, suggest the possibility of Cushing syndrome. Therefore, the correct diagnosis is A. Cushing syndrome.  

Cushing syndrome is a disorder that occurs due to the excessive production of cortisol, a hormone that regulates metabolism and stress response. The condition can cause a variety of symptoms, including weight gain, hirsutism, acne, hypertension, and menstrual irregularities or amenorrhea.  

Stein-Leventhal syndrome (B), also known as polycystic ovary syndrome, is a condition that affects the ovaries and can cause symptoms such as menstrual irregularities, hirsutism, and acne, but it typically does not cause the specific distribution of adipose tissue described in this case, and hypertension is not a common feature.   Turner syndrome (C) is a genetic disorder that affects females and can cause symptoms such as short stature, infertility, and amenorrhea, but it typically does not cause the specific distribution of adipose tissue, hirsutism, acne, or hypertension described in this case.  

Sheehan syndrome (D) is a condition that occurs due to pituitary gland damage during childbirth and can cause symptoms such as amenorrhea, lactation failure, and fatigue, but it typically does not cause the specific distribution of adipose tissue, hirsutism, acne, or hypertension described in this case.   Babinski-Froehlich syndrome (E), also known as hypothalamic hypogonadism, is a condition that affects the hypothalamus and can cause symptoms such as amenorrhea, infertility, and obesity, but it typically does not cause the specific distribution of adipose tissue, hirsutism, acne, or hypertension described in this case.  

Therefore, based on the symptoms and findings described, the correct diagnosis is A. Cushing syndrome. Treatment typically involves addressing the underlying cause of the condition, such as medication use or a tumor, and managing the symptoms with medications to reduce cortisol production and lifestyle modifications. It is important to manage the condition promptly to prevent potential complications and improve outcomes.


44. A 48-year-old patient was hospitalized with complaints of headache, nausea, wet cough, problematic breathing, impaired vision, excessive sweating, and salivation. He worked in a team that treated gardens with organophosphorus pesticides. In his blood: erythrocytes – 4.1 · 1012/L, Hb – 136 g/L, color index – 0.9, leukocytes – 13.0 · 109/L, ESR – 17 mm/hour. He was diagnosed with acute intoxication caused by organophosphorus pesticides. What is the most important diagnostic criterion o 922521I1 7 this pathology?

A. Decreased cholinesterase levels

B. Reticulocytosis

C. Leukocytosis

D. Thrombocytopenia

E. Anemia


Answer: Decreased cholinesterase levels

Explanation

The most important diagnostic criterion for acute intoxication caused by organophosphorus pesticides is decreased cholinesterase levels. Therefore, the correct answer is A. Decreased cholinesterase levels.   Organophosphorus pesticides inhibit the activity of cholinesterase, an enzyme that breaks down acetylcholine, a neurotransmitter that is essential for the proper function of the nervous system. The inhibition of cholinesterase leads to the accumulation of acetylcholine, which can cause a variety of symptoms, including headache, nausea, respiratory distress, vision changes, excessive sweating, and salivation.  

Reticulocytosis (B) is an increase in the number of immature red blood cells in the bloodstream, which may occur in response to anemia or other conditions, but it is not a specific diagnostic criterion for acute intoxication caused by organophosphorus pesticides.   Leukocytosis (C) is an increase in the number of white blood cells in the bloodstream, which may occur in response to infection, inflammation, or other conditions, but it is not a specific diagnostic criterion for acute intoxication caused by organophosphorus pesticides.  

Thrombocytopenia (D) is a decrease in the number of platelets in the bloodstream, which may occur in response to bleeding, medication use, or other conditions, but it is not a specific diagnostic criterion for acute intoxication caused by organophosphorus pesticides.   Anemia (E) is a decrease in the number of red blood cells or the amount of hemoglobin in the bloodstream, which may occur in response to bleeding, nutrient deficiencies, or other conditions, but it is not a specific diagnostic criterion for acute intoxication caused by organophosphorus pesticides.  

Therefore, based on the symptoms and findings described, the most important diagnostic criterion for acute intoxication caused by organophosphorus pesticides is decreased cholinesterase levels. Treatment typically involves the use of medications to counteract the effects of the pesticides, supportive care, and removal from further exposure. It is important to manage the condition promptly to prevent potential complications and improve outcomes.


45. A 55-year-old man on the 3rd day after an acute anterior myocardial infarction complains of shortness of breath and a dull pain behind the sternum that decreases when he leans forward. His blood pressure is 140/80 mm Hg and his heart sounds are muffled. ECG has shown atrial fibrillation with the ventricular rate of 110/min., pathological Q wave, and ST segment elevation in the right-sided chest leads. Make the right diagnosis:

A. Acute pericarditis

B. Dressler syndrome

C. Tietze syndrome

D. Pulmonary embolism

E. Dissecting aortic aneurysm


Answer: Acute pericarditis

Explanation

The symptoms and findings described in the patient, including shortness of breath, dull pain behind the sternum that decreases when leaning forward, muffled heart sounds, atrial fibrillation with a ventricular rate of 110/min, pathological Q wave, and ST segment elevation in the right-sided chest leads, suggest the possibility of acute pericarditis. Therefore, the correct diagnosis is A. Acute pericarditis.   Acute pericarditis is a condition in which the pericardium, the sac that surrounds the heart, becomes inflamed, leading to chest pain, shortness of breath, and other symptoms.

The condition can occur as a complication of a myocardial infarction, as well as other causes such as infection, autoimmune disorders, or trauma.   Dressler syndrome (B) is a type of pericarditis that occurs as a complication of a myocardial infarction, typically several weeks to months after the event, and can cause symptoms such as chest pain, fever, and pericardial effusion, but it typically does not cause atrial fibrillation or ST segment elevation.  

Tietze syndrome (C) is a condition in which the cartilage that connects the ribs to the breastbone becomes inflamed, leading to chest pain and swelling, but it typically does not cause muffled heart sounds, atrial fibrillation, or ST segment elevation.   Pulmonary embolism (D) is a condition in which a blood clot travels to the lungs and blocks blood flow, leading to symptoms such as shortness of breath, chest pain, and rapid heart rate, but it typically does not cause muffled heart sounds, atrial fibrillation, or ST segment elevation in the chest leads.  

Dissecting aortic aneurysm (E) is a condition in which the layers of the aortic wall separate, leading to a tear or rupture, and can cause symptoms such as severe chest pain, low blood pressure, and pulse deficits, but it typically does not cause muffled heart sounds, atrial fibrillation, or ST segment elevation in the chest leads.   Therefore, based on the symptoms and findings described, the correct diagnosis is A. Acute pericarditis. Treatment typically involves the use of medications to reduce inflammation and manage symptoms, as well as addressing any underlying causes of the condition. It is important to manage the condition promptly to prevent potential complications and improve outcomes.


46. The blood pressure and age were studied among 200 patients with essential hypertension. What statistical value can be used to measure the strength of the relationship between these characteristics?

A. Correlation coefficient

B. Representation error

C. Variation coefficient

D. Sigma deviation

E. Student’s t-test


Answer: Correlation coefficient

Explanation

The statistical value that can be used to measure the strength of the relationship between blood pressure and age among the 200 patients with essential hypertension is the correlation coefficient. Therefore, the correct answer is A. Correlation coefficient.   The correlation coefficient is a statistical measure that indicates the degree of association between two variables, such as blood pressure and age.

It is usually denoted by the symbol “r” and ranges from -1 to 1, where -1 indicates a perfect negative correlation, 0 indicates no correlation, and 1 indicates a perfect positive correlation between the variables.   In this case, the correlation coefficient can be calculated to determine the strength of the relationship between blood pressure and age among the 200 patients with essential hypertension. For example, a positive correlation coefficient would indicate that as age increases, blood pressure tends to increase as well, while a negative correlation coefficient would indicate the opposite.  

Representation error (B), variation coefficient (C), sigma deviation (D), and Student’s t-test (E) are not appropriate statistical values for measuring the strength of the relationship between blood pressure and age. The representation error refers to the difference between the true value and the estimated value of a parameter. The variation coefficient is a measure of relative variability. The sigma deviation is a measure of the spread or dispersion of a distribution. Student’s t-test is a statistical test used to compare the means of two groups.   Therefore, the correct answer is A. Correlation coefficient.


47. A 38-year-old woman, para 3, has a history of 5 artificial abortions. Five minutes after giving birth, she started bleeding from her genital tracts (350 mL). The patient’s condition is satisfactory, her pulse is 92/min., blood pressure is 100/60 mm Hg. There are no signs of placental expulsion, its manual removal has to be performed. In some spots the placenta is somewhat difficult to detach. Make the right diagnosis:

A. Partial placenta adherens

B. Trapped placenta

C. Total placenta adherens

D. Placenta accreta

E. Hypotonic bleeding


Answer: Partial placenta adherens

Explanation

Based on the given information, the most likely diagnosis for this patient is A. Partial placenta adherens.   Partial placenta adherens is a condition in which part of the placenta is abnormally attached to the uterine wall, making it difficult to detach after delivery. This can lead to postpartum hemorrhage and may require manual removal of the placenta.  

In this case, the patient experienced bleeding after giving birth and manual removal of the placenta was necessary. The fact that the placenta was somewhat difficult to detach in some spots suggests that there may have been some degree of placenta adherens.  

Trapped placenta (B) is a condition in which the placenta is not expelled after delivery and remains inside the uterus, leading to postpartum hemorrhage. Total placenta adherens (C) is a more severe form of placenta adherens in which the entire placenta is abnormally attached to the uterine wall, making it very difficult to detach. Placenta accreta (D) is a condition in which the placenta is abnormally attached to the uterine wall and may invade into the muscle, which can cause postpartum hemorrhage and may require surgical intervention.

Hypotonic bleeding (E) is a type of postpartum hemorrhage that occurs due to poor contraction of the uterus, which can lead to excessive bleeding.   Therefore, based on the symptoms and findings described, the most likely diagnosis for this patient is A. Partial placenta adherens. Treatment typically involves manual removal of the placenta and management of postpartum hemorrhage with medication and/or surgical intervention as needed. It is important to manage the condition promptly to prevent potential complications and improve outcomes.


48. A woman, who works at a pig farm, against the background of complete health developed sudden chills, fever of 39.9oC, intense headache, and nausea. The next day, she noticed a pain in her leg muscles and nosebleeds. On the 3rd day after the onset of the disease the woman’s condition is severe, her face is hyperemic, she has scleritis and subicteric sclerae. The liver is +3 cm. 24-hour urine output is 700 mL. Make the right diagnosis:

A. Leptospirosis

B. Yersiniosis

C. Influenza

D. Viral hepatitis

E. Hemorrhagic fever with renal syndrome


Answer: Leptospirosis

Explanation

Based on the given information, the most likely diagnosis for this patient is A. Leptospirosis.   Leptospirosis is a bacterial infection that is typically acquired through contact with the urine of infected animals, such as rats, mice, and pigs. The symptoms of leptospirosis can include fever, headache, muscle pain, nausea, vomiting, and in severe cases, liver and kidney damage.  

In this case, the patient works at a pig farm and developed sudden chills, fever, intense headache, and nausea, which are common symptoms of leptospirosis. The subsequent development of leg muscle pain and nosebleeds further supports this diagnosis. On the third day after the onset of the disease, the patient’s condition is severe, with hyperemic face, scleritis, subicteric sclerae, and an enlarged liver, which are all potential complications of leptospirosis.  

Yersiniosis (B) is a bacterial infection caused by the Yersinia bacterium, which can cause symptoms such as fever, abdominal pain, and diarrhea, but it is not typically associated with nosebleeds or liver and kidney damage.   Influenza (C) is a viral infection that can cause symptoms such as fever, headache, muscle aches, and fatigue, but it is not typically associated with nosebleeds or liver and kidney damage.   Viral hepatitis (D) is a type of liver inflammation caused by a viral infection, which can cause symptoms such as fever, fatigue, and jaundice, but it is not typically associated with nosebleeds or muscle pain.  

Hemorrhagic fever with renal syndrome (E) is a viral infection that can cause symptoms such as fever, headache, muscle pain, and kidney damage, but it is not typically associated with nosebleeds or liver enlargement.   Therefore, based on the symptoms and findings described, the most likely diagnosis for this patient is A. Leptospirosis. Treatment typically involves the use of antibiotics and supportive care to manage symptoms and prevent complications. It is important to manage the condition promptly to prevent potential complications and improve outcomes.


49. A 38-year-old man lives in an area that is endemic for echinococcosis. For the last 6 months he has been suffering from pain in his right subcostal region and fever. Echinococcal liver damage was suspected. What study will be the most informative in this case?

A. Ultrasound examination

B. Liver scan

C. Angiography

D. Biochemical testing

E. Survey X-ray-of the abdominal cavity


Answer: Ultrasound examination

Explanation

The most informative study in this case would be A. Ultrasound examination.   Echinococcosis is a parasitic infection caused by the tapeworm Echinococcus, and it can lead to the development of cysts in various organs, including the liver.

In this case, the patient is living in an area that is endemic for echinococcosis and is experiencing pain in the right subcostal region and fever, which are consistent with liver damage from echinococcosis.   Ultrasound examination is a non-invasive, low-cost imaging technique that is commonly used to diagnose and monitor liver damage from echinococcosis.

It can detect the presence of cysts, their location, size, and number, and can help to guide treatment decisions.   Liver scan (B) and angiography (C) are imaging tests that can be used to evaluate liver function and blood flow, but are not as sensitive as ultrasound for detecting echinococcal cysts.   Biochemical testing (D) can be used to assess liver function and to detect the presence of specific antibodies to echinococcus, but it is not as reliable as ultrasound for detecting cysts.  

Survey X-ray of the abdominal cavity (E) may reveal the presence of cysts, but it is not as sensitive as ultrasound for detecting small or deep-seated cysts.   Therefore, based on the information provided, the most informative study in this case would be an ultrasound examination of the liver to detect the presence of echinococcal cysts and to guide treatment decisions.


50. A 29-year-old woman was brought to the admission room with complaints of a sudden pain in her lower abdomen, weakness, and loss of consciousness. Her last menstruation was 6 weeks ago. Her pulse is 120/min., blood pressure – 80/50 mm Hg. There are pain and signs of peritoneal irritation low on the right side. In the blood: Hb – 106 g/L. Make the right diagnosis:

A. Interrupted tubal pregnancy

B. Ovarian torsion

C. Ovarian apoplexy

D. Acute appendicitis

E. Acute right-sided adnexitis


Answer: Interrupted tubal pregnancy

Explanation

Based on the given information, the most likely diagnosis for this patient is A. Interrupted tubal pregnancy.   Interrupted tubal pregnancy, also known as a ruptured ectopic pregnancy, is a potentially life-threatening condition in which a fertilized egg implants and begins to grow outside of the uterus, typically in the fallopian tube.

As the pregnancy progresses, the tube may rupture, leading to severe pain, internal bleeding, and shock.   In this case, the patient presents with sudden pain in the lower abdomen, weakness, loss of consciousness, tachycardia, hypotension, and signs of peritoneal irritation low on the right side, which are all consistent with a ruptured ectopic pregnancy. The fact that the patient’s last menstrual period was 6 weeks ago suggests that she may have been pregnant and the location of the pain on the right side is also suggestive of a tubal pregnancy.  

Ovarian torsion (B) is a condition in which the ovary twists on its own blood supply, causing severe pain, but it is not typically associated with peritoneal irritation or shock.   Ovarian apoplexy (C) is a condition in which a blood vessel in the ovary ruptures, causing sudden pain and bleeding, but it is not typically associated with peritoneal irritation or shock.  

Acute appendicitis (D) is a condition in which the appendix becomes inflamed and can cause severe abdominal pain, but it is typically located in the right lower quadrant and is not typically associated with peritoneal irritation or shock.   Acute right-sided adnexitis (E) is a condition in which the ovary and fallopian tube become inflamed, causing severe pain, but it is not typically associated with peritoneal irritation or shock.  

Therefore, based on the symptoms and findings described, the most likely diagnosis for this patient is A. Interrupted tubal pregnancy. Treatment typically involves emergency surgery to remove the affected tube and manage any internal bleeding. It is important to manage the condition promptly to prevent potential complications and improve outcomes.

 

Join the conversation
0% Complete